Show proof of point C in the given problem that involves Polar equation

  • #1
chwala
Gold Member
2,656
351
Homework Statement
See attached. My interest is on part C (Highlighted in yellow)
Relevant Equations
polar equations
1712784862585.png


c
1712784886609.png



Parts (a) and (b) are okay ... though the challenge was on part (a)

My graph had a plot of r on the y-axis vs θ on the x-axis). The sketch of my graph looks like is shown below;
1712793020305.png

I suspect the ms had θ on the x-axis vs r on the y-axis.

I used the equation ##r=\sqrt{\dfrac {1}{θ^2+1}}## with various values of ##θ## in the given domain and ended up with a graph opening on the right side of the first and fourth quadrant which is different from the attached graph from the mark scheme.

The shapes were similar.

1712785051115.png



Now for part (c), the steps are quite straightforward, I just want to check why they used ##r\sin θ##. Most probably its the distance of the maximum point of the graph from the x-axis.

The other working steps are quite clear- they used the quotient rule and then sign change to wrap up the question.

I am aware that cartesian to polar form we have ##x = r \cos θ## and ##y = r \sin θ## and therefore to determine distance in the ##y## direction one has to use ##y = r \sin θ##. If this answers my own query then thanks in advance.



1712785469242.png
 
Last edited:
Physics news on Phys.org
  • #2
I don't follow your graph of ##r^2 = \frac 1 {\theta^2 + 1}##. If ##\theta = 0##, then ##r = \pm 1##, so you would get points on the horizontal axis 1 unit to the right and 1 unit to the left. Also, if ##\theta = \pi/2##, the values of r would be ##\pm\frac 1 {\sqrt{(\pi/2)^2 + 1}}##, and not ##\pm 1## as you show on your graph.
chwala said:
I suspect the ms had θ on the x-axis vs r on the y-axis.
That's not the way that polar graphs work. Angles are measured in the CCW direction from the positive x axis (the ray ##\theta = 0##), and r is measured out along the ray defined by the angle.
 
Last edited:
  • Like
Likes chwala
  • #3
Mark44 said:
I don't follow your graph of ##r^2 = \frac 1 {\theta^2 + 1}##. If ##\theta = 0##, then ##r = \pm 1##, so you would get points on the horizontal axis 1 unit to the right and 1 unit to the left. Also, if ##\theta = \pi/2##, the values of r would be ##\pm\frac 1 {\sqrt{(\pi/2)^2 + 1}}##, and not ##\pm 1## as you show on your graph.
That's not the way that polar graphs work. Angles are measured in the CCW direction from the positive x axis (the ray ##\theta = 0##), and r is measured out along the ray defined by the angle.
Noted on the direction of r and the angle. Cheers.
 
Last edited by a moderator:
  • #4
Mark44 said:
I don't follow your graph of ##r^2 = \frac 1 {\theta^2 + 1}##. If ##\theta = 0##, then ##r = \pm 1##, so you would get points on the horizontal axis 1 unit to the right and 1 unit to the left. Also, if ##\theta = \pi/2##, the values of r would be ##\pm\frac 1 {\sqrt{(\pi/2)^2 + 1}}##, and not ##\pm 1## as you show on your graph.
That's not the way that polar graphs work. Angles are measured in the CCW direction from the positive x axis (the ray ##\theta = 0##), and r is measured out along the ray defined by the angle.
Mark, forgot your Latex after 16 years in PF???
 
Last edited by a moderator:
  • Haha
Likes chwala

Similar threads

  • Calculus and Beyond Homework Help
Replies
1
Views
934
  • Calculus and Beyond Homework Help
Replies
1
Views
866
  • Calculus and Beyond Homework Help
Replies
3
Views
537
  • Calculus and Beyond Homework Help
Replies
1
Views
474
  • Calculus and Beyond Homework Help
Replies
14
Views
303
  • Calculus and Beyond Homework Help
Replies
14
Views
3K
  • Calculus and Beyond Homework Help
Replies
5
Views
637
  • Calculus and Beyond Homework Help
Replies
1
Views
1K
  • Calculus and Beyond Homework Help
Replies
1
Views
848
  • Calculus and Beyond Homework Help
Replies
3
Views
584
Back
Top